Eu li errado, temos que lim x --> 0 f'
(x) = L. Assim,  a Regra de l' Hopital conforme mostrei demonstra que, de
fato, f'(c) = L.

Mas o que vc fez não mostra que f'(c) = L.

Artur Costa Steiner

Em Seg, 23 de abr de 2018 14:31, Igor Caetano Diniz <icaetanodi...@gmail.com>
escreveu:

> Se a questão tivesse um intervalo explícito [a,b] e diferenciável em todo
> ponto (a,b) exceto possivelmente num ponto c em (a,b) tal que lim f '(x) =
> L, x-> c, o que eu fiz estaria correto?
>
> 2018-04-23 14:11 GMT-03:00 Artur Steiner <artur.costa.stei...@gmail.com>:
>
>> Como f é contínua em 0, então, pela regra de L'Hopital,
>>
>> lim x --> 0+  (f(x) - f0))/(x - 0) = lim x --> 0+ f'(x) = L
>>
>> Pela definição de derivada lateral, o limite do primeiro membro é a
>> derivada à direita de 0. É só o que podemos concluir do enunciado. Nada
>> garante que a derivada à esquerda de 0 sequer exista.
>>
>> Artur Costa Steiner
>>
>> Em Dom, 22 de abr de 2018 22:45, Igor Caetano Diniz <
>> icaetanodi...@gmail.com> escreveu:
>>
>>> Boa noite,
>>> Gostaria de uma ajuda numa questão. Primeiro saber se pensei
>>> corretamente na maneira (1) e se é possível resolver como pensei também na
>>> maneira (2).
>>> Aí vai:
>>> Questão 5.3.8 do livro do Stephen Abbot, Understanding Analysis:
>>>
>>> Assuma que f é contínua em um intervalo que contém o zero e
>>> diferenciável em todo ponto diferente de zero. Se lim f ' (x) = L, x->0,
>>> prove que f ' (0) existe e é igual a L.
>>>
>>> O que pensei em fazer:
>>>
>>> Pensei em duas maneiras.
>>> 1)Se o limite existe em 0, então existem os limites laterais, limite a
>>> esquerda e limite a direita: lim x->0- f ' (x) = L e lim x->0+ f ' (x) = L.
>>> Lema: f ' (c) = lim f(c+h)-f(c-h)/2h = lim [ f(c+h)-f(c) +f(c) - f(c-h)
>>> ]/2h = 1/2 lim x->c-[f(c+h)-f(c)/h] + 1/2 lim x->c+ [f(c+h)-f(c)/h]
>>>
>>> Logo como existem esses limites laterais, existe a derivada em 0, e
>>> portanto, é L
>>>
>>> 2) queria tentar fazer, usando uma sequência xn<0 com limxn = 0 e yn>0
>>> com lim yn = 0 e provar que lim(f(yn)-f(xn)/(yn-xn)) = f'(0) = L. Mas sinto
>>> que isso é verdade e não sei provar
>>>
>>> Abraços
>>>
>>> --
>>> Esta mensagem foi verificada pelo sistema de antivírus e
>>> acredita-se estar livre de perigo.
>>
>>
>> --
>> Esta mensagem foi verificada pelo sistema de antivírus e
>> acredita-se estar livre de perigo.
>>
>
>
> --
> Esta mensagem foi verificada pelo sistema de antivírus e
> acredita-se estar livre de perigo.

-- 
Esta mensagem foi verificada pelo sistema de antiv�rus e
 acredita-se estar livre de perigo.

Responder a